a Putnam-exam-level system of equations

Sdílet
Vložit
  • čas přidán 29. 06. 2024
  • This question is B3 from the 1969 Putnam exam. Very suitable for calculus 2 students.
    Subscribe czcams.com/users/blackpenredpe...
    Support: / blackpenredpen
    Shop: teespring.com/stores/blackpen...
    blackpenredpen

Komentáře • 210

  • @leadnitrate2194
    @leadnitrate2194 Před 4 lety +193

    Legend has it that the bunny can solve the Reimann Hypothesis in four different ways.

    • @noahzuniga
      @noahzuniga Před 4 lety +23

      blackpenredpen video in 2024: How to solve the Riemann Hypothesis (two ways), Calc 3 Practice

    • @lucaschai5788
      @lucaschai5788 Před 4 lety +5

      是吗?Are you sure?

    • @MarkusDarkess
      @MarkusDarkess Před 4 lety +5

      Twin primes will be found at multiples of 15.
      Odd numbers will find twin primes ending in 1,3 and 7,9
      Even numbers will find twin primes 9,1.
      Euclid PNT Q=P1,P2,P3...... P24
      First three primes are 2,3,5=30.
      So multiplying any prime after will only find spots at intervals 30.
      This is only partial proof of the riemann hypothesis. I have the full proof written down.
      But no friends or family want to talk about it... because to them math is pointless. And went I went to a college the professor wasn't there and when they asked what it was about. I told them it was essentially a thesis... but after they found out I had no college. They said it was probably nothing before even looking.
      I don't have the skills to take it even higher because I cannot afford college. And I have been working on this for two years alone. But if I go to college or not won't take away the meaning of this proof.
      And after comprehending the full scope of this. I am going to post my
      I just want to be a productive member of society and have a voice. This is why many mathematicians die before they are recognized.
      I would really like a response before I post it. But... I will probably post it within the next 24hours.

    • @MarkusDarkess
      @MarkusDarkess Před 4 lety +2

      @@noahzuniga I guess I am the quantum bunny
      I am the last mimzy. (A movie) I have the other 4 proofs too

    • @user-mt9ux2di6u
      @user-mt9ux2di6u Před 3 lety +1

      Hahaha!

  • @sashabell9997
    @sashabell9997 Před 4 lety +71

    2:54 "sometimes he likes to be on the bottom, sometimes he likes to be on the top"
    I guess you could say the variable is pretty versatile then huh?

  • @Maou3
    @Maou3 Před 4 lety +23

    Algebruh moment at the end there

  • @aronpacino8009
    @aronpacino8009 Před 4 lety +41

    5:55
    I'm sensing a double factorial here, not yet sure, i'm not yet done watching

    • @blackpenredpen
      @blackpenredpen  Před 4 lety +11

      You were right!

    • @internetbad3575
      @internetbad3575 Před 4 lety +3

      Wow you must have like 200 IQ to have seen that coming

    • @aronpacino8009
      @aronpacino8009 Před 4 lety +2

      @@blackpenredpen yes, it seemed to be so, i noticed that they are just like a factorial, but instead of decreasing by 1, they are decreasing by 2. Hehehe

    • @rahimeozsoy4244
      @rahimeozsoy4244 Před 4 lety +1

      Are you 140 IQ

  • @bingcheng8419
    @bingcheng8419 Před 4 lety +8

    You should check limit of a_(2n+1)/a_(2n+2), in case the limits are not same.

  • @integralboi2900
    @integralboi2900 Před 4 lety +12

    Yay! You finally uploaded!

    • @blackpenredpen
      @blackpenredpen  Před 4 lety +11

      Sorry, took too long to decide “which mic” to use. : )

  • @Meetu4151
    @Meetu4151 Před 4 lety +1

    Hi bro!! Love your maths videos, they keep me motivated!!!

  • @Rahulsingh-lw8hk
    @Rahulsingh-lw8hk Před 4 lety +3

    i love these types of problems thanks to you to make me able to see fun in mathematics!

  • @Nerketur
    @Nerketur Před 4 lety +4

    ::confused face:: "square root... pi/2???"
    ::long pause::
    ::cut to real solution:: "its obviously plus or minus the square root of 2/pi!"
    thank you for that. i really needed that laugh. xD
    this truly made my day. :D

  • @MathemaTeach
    @MathemaTeach Před 4 lety +2

    Nice microphone, i wanna use one like that in my math videos too..
    This is a great help for students on lock down but still needing to learn...

  •  Před 4 lety +3

    Just found out about your channel! I love math

  • @andrewmccauley6262
    @andrewmccauley6262 Před 11 měsíci

    Did it via a bunch of factorials and stirling's approximation, was very satisfying cancelling everything out to get to the right answer!

  • @andersonabelriquelmefernan8725

    Genial!!, como siempre.
    Me encantan tus videos, a pesar de no hablar tu mismo idioma entiendo la mayor parte del video. Muchas gracias y exitos.
    Psd: me gustaría que añadas subtítulos al español :)

  • @erickherrerapena8981
    @erickherrerapena8981 Před 4 lety +1

    Jajajaja no me esperaba que pusieras tu pequeño error de cálculo, pero quedó bien. ¡¡Buen video!!

  • @killergames7923
    @killergames7923 Před 4 lety +3

    Hi! I've been watching your videos for weeks and I am really fascinated. I love math and I really want to study it at advanced level and be good at it, but I don't know how to begin. Have you got any advices(maybe some videos or sites or books)?
    I hope you read this message
    Thank you, you're amazing

  • @gabest4
    @gabest4 Před 4 lety +3

    Printed the numbers up to a1000 and the solution is almost certainly correct. Good job! Although the convergence of an/an+1 to 1 is not mononically decreasing. Around a93 it starts oscillating a little. Could be some strange floating point error.

  • @egillandersson1780
    @egillandersson1780 Před 4 lety +7

    I recognized the Wallis product only at 9:17 . Shame on me !
    Very interesting video. I didn't think that pi woud come here !

    • @blackpenredpen
      @blackpenredpen  Před 4 lety +7

      pi always like to come to math parties

    • @FaranAiki
      @FaranAiki Před 4 lety

      It's hard to remember some formula :D

    • @lucaschai5788
      @lucaschai5788 Před 4 lety +3

      @@blackpenredpen pi always come to parties that he shouldn't go :D

    • @egberthuizinga4408
      @egberthuizinga4408 Před 4 lety

      @@blackpenredpen i always like to come to math parties

  • @tusharbansal8794
    @tusharbansal8794 Před 4 lety +1

    A big fan sir

  • @aarl901
    @aarl901 Před 4 lety

    I have found by myself, but it took me quite a long time. finding explicit formulas for a(2n+2) and a(2n+3) with factorials, then using Stirling Formula and many calculus manipulations. I didn't remember Wallis Product, clearly it would help me find faster the answer

  • @pkmath12345
    @pkmath12345 Před 4 lety +9

    Series! Cool haha I like to cover topic related to the patterns, also like that lil thing you are holding~ great

  • @MathsCoach
    @MathsCoach Před 4 lety +19

    Hi. Where do you buy your microphone covers from? They look so cool!

    • @krish1349
      @krish1349 Před 4 lety +4

      That's a pokeball, with asH inside it😂

    • @MathemaTeach
      @MathemaTeach Před 4 lety

      I want one too... 😊

  • @gustavopaz5453
    @gustavopaz5453 Před 4 lety

    Long time no see, sir!

  • @agraneesingh755
    @agraneesingh755 Před 4 lety

    phenomenal 🙀🙀

  • @nafrost2787
    @nafrost2787 Před 3 lety +1

    This video, is why the pie notation for multiplication was invented.

  • @santoshmishra5993
    @santoshmishra5993 Před 2 lety +1

    our blckpenredpen will be doing goldback conjecture also in 2 methods till 2025

  • @soumyanamdeo2347
    @soumyanamdeo2347 Před 4 lety +5

    Sir you are awsome

  • @kinshuksinghania4289
    @kinshuksinghania4289 Před 4 lety

    Great ✌️✌️, I was suspecting π right from the start!!!

  • @saharhaimyaccov4977
    @saharhaimyaccov4977 Před 4 lety

    Amazing..

  • @LaBrantMath
    @LaBrantMath Před 4 lety +14

    I need a rabbit like that for when I get stuck explaining to my students.

  • @omarelric
    @omarelric Před 4 lety

    AMAZING!!!!! That was delightful, how can you deny math’s platonic qualities when you glance at stuff like this?

  • @DrMasi
    @DrMasi Před 4 lety

    AWESOME

  • @POLKQWER
    @POLKQWER Před 4 lety +2

    6:06 may i ask what is he calling that term? can't quite hear it well.

    • @John-uh1pb
      @John-uh1pb Před 4 lety +2

      "The limit as n goes to infinity of a-whatever over a-sub whatever (the same whatever) and then plus one"

  • @madmuffin2511
    @madmuffin2511 Před 4 lety +1

    Where is the error in the following ?:
    a_n*a_n+1 = n
    --> a_n = n/a_n+1
    multiply with a_n and use a_n/a_n+1 --> 1 for n --> infty
    --> a_n**2 = n*a_n/a_n+1 --> n
    --> a_n --> sqrt(n) (*)
    a_1 = (n-2)!! / (n-1)!! *a_n --> sqrt(2/pi) * sqrt(n) --> infty with (*) and square root of wallis product.

  • @cameronkhanpour3002
    @cameronkhanpour3002 Před 4 lety +10

    hahaha that part at the end was funny "底下 底下 底下"

  • @warlokyx
    @warlokyx Před 4 lety

    a1 is being a chaotic vers and I'm here for it 😂

  • @Steven-ov4no
    @Steven-ov4no Před 3 lety +1

    曹老師的頻道越來越多朋友了🤣

    • @Steven-ov4no
      @Steven-ov4no Před 3 lety +2

      13:55 (發現怪怪)
      13:57 等一下等一下!
      應該要找小兔兔來幫忙才對

    • @blackpenredpen
      @blackpenredpen  Před 3 lety +1

      哈哈哈 是啊

    • @aashsyed1277
      @aashsyed1277 Před 3 lety

      @@blackpenredpen WHY??????????????????

  • @gopalans7599
    @gopalans7599 Před 4 lety

    @blackpenredoen
    Could you pl find the limit of x tending to infinity
    Root(x^2+2x)-x

  • @gardenmenuuu
    @gardenmenuuu Před 4 lety

    sir I am your great fan n regular viewer.Where are you from? whats ur job ,qualifications i wanna know plzzz

  • @ronandodds
    @ronandodds Před 4 lety +2

    stupid question, why is the (2n-2) squared in the equation outlined in red? Shouldn't it just be to the first power times (2n) ?

    • @davidgould9431
      @davidgould9431 Před 4 lety +2

      We are multiplying all the evens up to (2n - 2) with 2 x 4 x 6 x .... x (2n - 6) x (2n - 4) x (2n - 2) x 2n, so the (2n - 2) is squared, too.

  • @Mathematica1729
    @Mathematica1729 Před 4 lety +2

    Please somebody explain me at 7:58 why did he put the square on (2n-2) in the denominator.

    • @davidgould9431
      @davidgould9431 Před 4 lety +4

      Between 2, 4, 6, ... 2n there is a (2n - 2).
      Edit: ok, that was a bit terse.
      We are multiplying (2 x 4 x 6 x ... x (2n - 2)) with (2 x 4 x 6 x ... x 2n). Now, 2n is bigger than 2n - 2. You match up all the terms to 2n - 2, so they get squared. There is still one term 2n left, so multiply that in too.

  • @z4zuse
    @z4zuse Před 4 lety

    Nice to see Nijntje came to the rescue

  • @VaradMahashabde
    @VaradMahashabde Před 4 lety +4

    My guess : alternatively divide and mulitply, wallis product style,
    so a_1 = a_1 * a_2 / (a_2 * a_3) * (a_3 * a_4) ... =1 /2 * 3 / 4 ...
    = lim_{n -> inf} (2n+1)!! / (2n)!! or = lim_{n -> inf} (2n-1)!! / (2n)!!
    But the limit is infinite for the left one and zero for the right one 🤔

    • @valeriobertoncello1809
      @valeriobertoncello1809 Před 4 lety

      I also thought about this method of solving it, though I couldn't find a way to include the condition imposed by the limit in the proof!
      If you square both sides in your last expression you should get:
      (a_1)² = 1²×3²×5²×•••/2²×4²×6²×••• = 2/π
      and therefore a_1 = ±√(2/π)
      But, again, how does the limit come into play in this proof? must be wrong :c

    • @angelmendez-rivera351
      @angelmendez-rivera351 Před 4 lety

      Varad Mahashabde You are probably not carrying out the product correctly. You are supposed to ultimately just have a ratio of double factorials multiplied by some extra normalizing factor.

    • @angelmendez-rivera351
      @angelmendez-rivera351 Před 4 lety

      Also, I am not quite sure that your claim that lim (2n + 1)!!/(2n)!! (n -> ♾) is true.
      EDIT: The claim is true, but irrelevant, since a(1)^2 = lim (2m + 1)!!(2m - 1)!!/(2m)!!^2 (m -> ♾), it is not the case that a(1) = lim (2m - 1)!!/(2m)!! (m -> ♾).

    • @angelmendez-rivera351
      @angelmendez-rivera351 Před 4 lety

      Valerio Bertoncello The idea is that lim a(n)/a(n + 1) (n -> ♾) = 1 implies that 1/a(1)^2 = lim (2m)!!^2/[(2m - 1)!!(2m + 1)!!] (m -> ♾) .

  • @nikitakipriyanov7260
    @nikitakipriyanov7260 Před 3 lety

    I solved this myself and got the same answer! Despite the fact I started to forget all this math (and now trying to remember it as a way to massage my brain, while others solve sudokus and crosswords).

  • @haltyouropinions3780
    @haltyouropinions3780 Před 4 lety

    at first i was confused it looks to me like at the end of the wallis product it would be
    (2n-1)/(2n-2) * (2n-1)/(2n) but then what about the other (2n-2)?
    well before the (2n-1) term, there is (2n-3), so the other (2n-1) goes to that.

  • @Sam-zx4rp
    @Sam-zx4rp Před 4 lety +3

    Nice year, btw.

  • @joekerr5418
    @joekerr5418 Před 4 lety

    Shouldn't there be a (2n-1) before 2n at 8:21?

  • @user-bz2qm2ij9d
    @user-bz2qm2ij9d Před 4 lety

    Nice.

  • @guest_of_randomness
    @guest_of_randomness Před 4 lety +14

    🤣“等一下”

    • @user-oe5eg5qx4c
      @user-oe5eg5qx4c Před 4 lety

      0:33 and don't worry ,if I get stuck 小兔兔 will help us out

  • @Vampianist3
    @Vampianist3 Před 4 lety +2

    So in the end you asked 小兔兔 for help right?

  • @l_szabi
    @l_szabi Před 2 lety

    13:50 that is such a math teacher moment :D

  • @msolec2000
    @msolec2000 Před 4 lety +1

    But what if you had started with lim (a_(2n-1)/a_(2n)) instead?

  • @duggydo
    @duggydo Před 4 lety

    Did you sing the song at the end bprp?

    • @erikkonstas
      @erikkonstas Před 4 lety

      It says on the bottom-right "outro song by Zach & Jonah (check out MotorMusic)" very clearly, so...

  • @soumyanamdeo2347
    @soumyanamdeo2347 Před 4 lety +1

    How can I send you questions ???

    • @blackpenredpen
      @blackpenredpen  Před 4 lety +3

      Twitter is the best place since u can just tweet me a pic.

  • @Risu0chan
    @Risu0chan Před 2 lety

    I was expecting a limit of a[n]. You can show an equivalence a[n] ~ √n , that is limit a[n]/√n = 1

  • @allmight801
    @allmight801 Před 4 lety +1

    gotta solve 'em all

  • @nitayderei
    @nitayderei Před 4 lety +2

    1969. Nice.

  • @maxamedmuuse4882
    @maxamedmuuse4882 Před 4 lety

    Great

  • @hach1koko
    @hach1koko Před 4 lety

    the fact that lim a_(n)/a_(n+1) = 1 is just a consequence of the definition, right ? It's easy to see that a_(2n)/a_(2n+2) and a_(2n+1)/a_(2n+3) both converges to 1 by using the definition of a_(n).

  • @miruten4628
    @miruten4628 Před 4 lety +1

    You should confirm that also lim_{n->∞} [a_{2n-1} / a_{2n}] = 1 with those values of a_1. Otherwise there would be no solution for a_1.

  • @hamsterdam1942
    @hamsterdam1942 Před 4 lety

    +-sqrt(pi/2) could be also an answer

  • @andraspongracz5996
    @andraspongracz5996 Před 4 lety

    We are NOT done: you have only demonstrated that if there is an answer, then it must be $\pm \sqrt{2/\pi}$. The problem is that you only computed the limit of a subseries, namely the one obtained by taking every other term of $a_n/a_{n+1}$. You can easily finish the proof by showing the same result for the subseries of the remaining elements. (Or an elegant finish: it is clear from the definition of the $a_i$ that $a_n/a_{n+2}=n/(n+1)$ tends to 1, so if the above half of the quotient series tends to 1, then the other half does too.)

    • @angelmendez-rivera351
      @angelmendez-rivera351 Před 4 lety

      András Pongrácz This is a moot point. The limit of the ratio of the subsequence is the same as the limit of the ratio of the sequence itself. This is because if lim a(n)/a(n + 1) = 1, then n = 2m, and as n -> ♾, m -> ♾. Since this transformation is bijective, which is trivial, the implication holds in the other direction.

    • @andraspongracz5996
      @andraspongracz5996 Před 4 lety

      @@angelmendez-rivera351 The limit of a subsequence is the same as the limit of the sequence... provided that the sequence has a limit. It is not a moot point, but a theoretical mistake (which is of course easy to fix, as I wrote). Imagine that every other term is 1, and every term wit odd index $n$ is $n$. If you only focus on the even indices, you might think that the limit is 1.

    • @angelmendez-rivera351
      @angelmendez-rivera351 Před 4 lety

      András Pongrácz Okay, but we already knew the sequence has a limit at all, it is literally part of the premise of the problem. So, it is indeed a moot point. There is nothing theoretically incorrect that he did in this video. He didn't use induction to prove the form of the terms a(2k) and a(2k - 1), but to be fair, proving it is so trivial, it's not as if anyone watching the video is confused about why he didn't do it.

  • @grawuka6900
    @grawuka6900 Před 4 lety

    A bit sad the derivation isn't done from the Stirling formula as it's what is expected to be known from Putnam candidates.
    Still a nice video :)

    • @angelmendez-rivera351
      @angelmendez-rivera351 Před 4 lety

      Grawuka I think the Wallis product is better known than the Stirling asymptotic approximation formula. In any case, the Stirling approximation can be derived from the Wallis product, and the Wallis product can be derived from the Stirling approximation, hence it really does not matter.

  • @Happy_Abe
    @Happy_Abe Před 4 lety

    Wouldn't the higher degree of n in the denominator cause the limit to go to 0?

    • @angelmendez-rivera351
      @angelmendez-rivera351 Před 4 lety

      Avraham Radin The denominator does not have a higher degree of growth with respect to n. Just to be precise, a(1)^2 = lim (2n - 1)!!(2n + 1)!!/(2n)!! (n -> ♾), which is the reciprocal of the Wallis product. (2n - 1)!!(2n + 1)!! and (2n)!!^2 have the same growth. This can be proven by using double factorial identities to convert into factorials and exponentials, and then compare the numerator and denominator by using the Stirling asymptotic approximation.

  • @josedearimatea4832
    @josedearimatea4832 Před 4 lety

    Shouldn't the pi over 2 be squared on that last part?

  • @yasharya9765
    @yasharya9765 Před 4 lety +1

    @blackpenredpen today I uploaded a Putnam 1992 functional equation problem try it

  • @Karimkarim-mr2lf
    @Karimkarim-mr2lf Před 4 lety

    Wow 😲😲

  • @rosamondcarson1508
    @rosamondcarson1508 Před 4 lety +1

    哈哈哈,,,我听到了“等一下等一下”

  • @criskity
    @criskity Před 4 lety +5

    13:56 Chinese? 等一下!

  • @Geffde
    @Geffde Před 4 lety

    There’s gotta be more solutions than that because a1 = a2 = 1 also generates the same set of answers...

    • @niklasschmidt3610
      @niklasschmidt3610 Před 4 lety

      You may be able to construct a series with a_n * a_(n+1) =n with a_2k and a_(2k+1) like the ones in the video with (a_1)^2 =1, but then the lim (a_n/a_(n+1)) = 2/pi and not 1. So I don't think there are different answers. I'm actually sure there are none

  • @nicolascamargo8339
    @nicolascamargo8339 Před 10 měsíci

    Wow ese producto está curioso

  • @divisix024
    @divisix024 Před 8 měsíci

    13:56 「等一下等一下等一下」wwwww

  • @krishnasimha8097
    @krishnasimha8097 Před 4 lety +4

    It shows a pattern=n!!/n-1!!

    • @FaranAiki
      @FaranAiki Před 4 lety +1

      Don't forget the square :D

    • @angelmendez-rivera351
      @angelmendez-rivera351 Před 4 lety

      Not n!!/(n - 1)!!, but (2n)!!/(2n - 1)!!, and you must square the ratio.

  • @Tomaplen
    @Tomaplen Před 4 lety

    13:45 Me on every single fucking test... u.u

  • @djvalentedochp
    @djvalentedochp Před 4 lety

    Pi always being on strange places

    • @angelmendez-rivera351
      @angelmendez-rivera351 Před 4 lety

      DJ VALENTE DO CHP It isn't as strange as it seems. There are a bunch of double factorials in this product, and the double factorials come from the Gamma function being evaluated at half integers, which is equivalent to integrating a cylindrically symmetric bell curve. The cylindrical symmetry is where π becomes relevant.

  • @brownwater6212
    @brownwater6212 Před 4 lety +3

    Love the pokeball mic.

  • @OfficialGamingNetwork
    @OfficialGamingNetwork Před 4 lety

    2:54 - 2:58 mood

  • @domanicmarcus2176
    @domanicmarcus2176 Před 4 lety

    Can you do a video on washers on disks? Can you please do the same problem in both washers and discs? I have a problem with setting it up. Can you please do more than just 1 problem? Please explain in your video why 1 method would be easier than the other method. Please, explain the setup. Also, can you please do the same problem if it was rotated about the x-axis and then about the y-axis. In some problems, I must solve for x and in other problems and in other problems I must solve for y. Can you please explain when to solve for x or y? I have a problem with setting them up. You don't need to solve the problems. Just the setup is where I am running into a problem. I can integrate the problem(s) myself. The integration is the easy part. The setup is the HARD PART. Thank You, Sir.

  • @Aria-mm9fq
    @Aria-mm9fq Před 4 lety

    Have you ever heard anything about maryam mirzakhani?

  • @guest_of_randomness
    @guest_of_randomness Před 4 lety

    i should have known that he will come out with a crazy weird formula in the end. but then i clicked in... and saw a weird formula...

  • @markkennedy9767
    @markkennedy9767 Před 4 lety +1

    It seems obvious but can anyone tell me why
    if the lim (a(n)/a(n+1))=1,
    then the lim(a(2n)/a(2n+1))=1.
    Otherwise I may have been able to do this

    • @stephenbeck7222
      @stephenbeck7222 Před 4 lety +1

      Let u=2n. Then as n goes to infinity, u also goes to infinity. The limit with the 2n and 2n+1 becomes a(u)/a(u+1), which is equivalent to the given limit in the question.

    • @rajawi9999
      @rajawi9999 Před 4 lety

      @@stephenbeck7222 that's incorrect because a(n) isn't a function

    • @rajawi9999
      @rajawi9999 Před 4 lety

      @@stephenbeck7222 that's incorrect, what if a(n+1)=n and a(n) =1, we have here a(n+1)*a(n)= n but a(n)/a(n+1)=>0

  • @Magistrixification
    @Magistrixification Před 3 lety

    Do you ever feel like you are inadequate?

  • @Kdd160
    @Kdd160 Před 4 lety +2

    He always starts with
    "Hello let's do some math for fun"

  • @xalluniverse9028
    @xalluniverse9028 Před 4 lety

    It's interesting that a1 = 0.5!

  • @bigwatermelon2973
    @bigwatermelon2973 Před 4 lety +3

    13:56 等一下,等一下 lol

  • @bttfish
    @bttfish Před 4 lety

    For the sake of rigor, you need to construct a concrete sequence to prove that sqrtpi/2 can actually be a1

    • @blackpenredpen
      @blackpenredpen  Před 4 lety

      What did you mean by “prove that sqrt(pi/2) can actually be 1”?

    • @angelmendez-rivera351
      @angelmendez-rivera351 Před 4 lety

      What do you mean? The infinite product is the limit of a concrete well-known sequence.

  • @arandompersononyoutube3739

    Is it pokeball or mic ?

  • @DanBurgaud
    @DanBurgaud Před 3 lety

    13:40 LOL! you took a shortcut skipping a step ending with 13:50

  • @pilime103
    @pilime103 Před 4 lety

    等一下XD 老師是哪裡人啊

    • @blackpenredpen
      @blackpenredpen  Před 4 lety

      我來自台灣喔

    • @pilime103
      @pilime103 Před 4 lety

      我是高中數學老師,關注你的頻道一段時間了,本來以為你是ABC,原來是台灣人啊~老師應該是在大學任教吧

  • @ameerunbegum7525
    @ameerunbegum7525 Před 4 lety

    I think there is pikachu in that ball

  • @gaaraofddarkness
    @gaaraofddarkness Před 4 lety

    13:52. Hug Diya na!! lol

  • @yaleng4597
    @yaleng4597 Před 4 lety +2

    等一下,差點搞砸了
    LMAO

  • @gorillajock
    @gorillajock Před 2 lety

    That's Wallis product, not Walli's product

  • @F1U7R2Y9
    @F1U7R2Y9 Před 4 lety +2

    Plz do a collab on a topic of math with muPrimeMath. plzzzzzzzzzzzzzzzzzzzzzzzzzzzzzzzzzzzzzzzzzzzzzzzzzzzzzzzzzzzzzzzzzzzzzzzzzzzzzzzzzzzzzzzzzzzzzzzzzzzzzzzzzzzzzzzzzzzzzzzzzzzzzzzzzzzzzzzzzzzzzzzzzzzzzzzzzzzzzzzzzzzzzzzzzzzzzzzzzzzzzzzzzzzzzzzzzzzzzzzzzzzzzzzzzzzzzzzzzzzzzzzzzzzzzzzzzzzzzzzzzzzzzzzzzzzzzzzzzzzzzzzzzzzzzzzzzzzzzzzzzzzzzzzzzzzzzzzzzzzzzzzzzzzzzzzzzzzzzzzzzzzzzzzzzzzzzzzzzzzzzzzzzzzzzzzzzzzzzzzzzzzzzzzzzzzzzzzzzzzzzzzzzzzzzzzzzzzzzzzzzzzzzzzzzzzzzzzzzzzzzzzzzzzzzzzzzzzzzzzzzzzzzzzzzzzzzzzzzzzzzzzzzzzzzzzzzzzzzzzzzzzzzzzzzzzzzzzzzzzzzzzzzzzzzzzzzzzzzzzzzzzzzzzzzzzzzzzzzzzzzzzzzzzzzzzzzzzzzzzzzzzzzzzzzzzzzzzzzzzzzzzzzzzzzzzzzzzzzzzzzzzzzzzzzzzzzzzzzzzzzz

    • @blackpenredpen
      @blackpenredpen  Před 4 lety +1

      We will! Maybe after a few weeks tho since he has AP exams and I have other things to work on as well.

    • @F1U7R2Y9
      @F1U7R2Y9 Před 4 lety

      @@blackpenredpen o' Thanks for reply.

  • @rajawi9999
    @rajawi9999 Před 4 lety

    you should prove first the relation that you wrote betwen a2n+1 and n

  • @whybeee
    @whybeee Před 4 lety

    So all the jokes about the Pokéball are now true

  • @rhemalogos7980
    @rhemalogos7980 Před 4 lety

    Is every thing equal to pii or what? crazy as pii followed us since primary school

    • @criskity
      @criskity Před 4 lety

      Pi pops up in the oddest places. But as 3Blue1Brown says, wherever you find pi, a circle is involved somehow. It may be hidden, but it's there.

    • @angelmendez-rivera351
      @angelmendez-rivera351 Před 4 lety

      CNVideos Yes. In this case, the sequence {a(n)} is a ratio of double factorials, which can all be expressed in terms of ratios of products of fractional factorials, which are the result of the n-volume of the bell curve, which has polar properties. In those polar properties is where the circle is hidden.

  • @user-go6dj6lg8b
    @user-go6dj6lg8b Před 2 lety +1

    13:56 等一下XD 突然冒出中文我嚇到

  • @newmansLP
    @newmansLP Před 4 lety

    Didn't you just proof that for even/odd? If an is odd and an+1 is even the answer would be root(pi/2) right? correct me if I'm wrong.

    • @angelmendez-rivera351
      @angelmendez-rivera351 Před 4 lety

      Goatgod No. lim a(n)/a(n + 1) (n -> ♾) = lim a(2n)/a(2n + 1) (n -> ♾). This is the property the question is reliant on.

  • @MarcDM92
    @MarcDM92 Před 4 lety +1

    Just about to comment "Hey you missed to square Wallis Product result due to the squares of the limit formula..." but NO NO!! DON'T DO IT!!!! :)

    • @LLWN84
      @LLWN84 Před 4 lety +1

      I was too, but later realised. Lol

  • @emlmm88
    @emlmm88 Před 4 lety

    But can you have an uncountable system of equations? ... that's underspecified?